Hello, dear friend, you can consult us at any time if you have any questions, add WeChat: daixieit

ECMT6006 Applied Financial Econometrics

2022S1

Final Exam Questions

1    Main exam

1.1    Multiple-answer questions (12 points)

1.  Select the incorrect statement(s) about asset returns.

(a)  Log return is the logarithm of gross return, and it is a gross return concept.  (incorrect) (b)  Another name of log return is continuously compound return.  (correct)

(c)  The multi-period gross return is the sum of the single-period gross returns.  (incorrect)

(d)  The portfolio arithmetic net return can be written as a value-weighted average of the arith- metic net returns of individual assets in the portfolio.  (correct)

(e)  For the same amount of deposit at the beginning of the year and the same per annum

interest rate, if the bank pays interest rate more often over the year, then the more return you will earn from the deposit at the end of the year.  (correct)

2.  Select the correct statement(s):

(a)  A martingale difference sequence is necessarily a zero mean white noise process.  (correct)

(b)  A martingale difference sequence is necessarily a white noise process but does not necessarily

have zero mean.  (incorrect)

(c)  An i.i.d. sequence is not necessarily a white noise sequence.  (incorrect)

(d)  A zero mean i.i.d. sequence is necessarily a martingale difference sequence.  (correct)

(e)  The variables in a martingale difference sequence is necessarily mutually independent.  (in-

correct)

3.  Consider the following regression:

Yt  = β0 + β1 Xt1 + β2 Xt2 + β3 Xt3 + et .

Suppose that we want to test the following null hypothesis:

H0  : β 1  = β2  = β3  = 0.

The alternative hypothesis of this null hypothesis is:

(a)  β 1   β2   β3   0

(b)  β 1   0 and β2   0 and β3   0

(c)  β 1   0 or β2   0 or β3   0 (correct)

(d)  β 1   β2  or β2   β3  or β 1   β3

4.  Let  {Yt } be a time series and Ft  be the information set which contains the information of Yt ,Yt1 , . . .. Define a new time series {Xt } such that

Xt  = log(Yt ) E[log(Yt )|Ft1].

Select the correct statement(s) about {Xt }.

(a)  {Xt } is a white noise.  (correct)

(b)  {Xt } is a martingale difference sequence.  (correct)

(c)  {Xt } is an i.i.d. sequence.  (incorrect)

(d)  {Xt } is serially correlated.  (incorrect) (e)  {Xt } has nonzero mean.  (incorrect)

5.  Please select the correct statement(s) about volatility forecast and model selection.

(a) We can use a t test to perform in-sample model selection between a GJR-GARCH(1,1)

model and a GARCH(2,1) model.  (incorrect)

(b) If we use QLIKE loss instead of the quadratic loss to quantify the loss of a volatility forecast,

then the optimal point forecast of the volatility will be different.  (incorrect)

(c)  Mean squared forecast error (MSFE) is not the only criterion that can be used when we use out-of-sample analysis to perform volatility model selection.  (correct)

(d)  Diebold-Mariano test can be used to test the optimality of the forecast produced by a volatility model.  (incorrect)

(e)  The HQIC imposes lighter penalty on each additional parameter in the model than AIC.

(incorrect)

6.  Please select the correct statement(s).

(a)  The daily integrated variance is an ex-ante risk measure.  (incorrect)

(b) We don’t need a parametric model to obtain the realized variance estimate.  (correct)

(c)  The conditional variance of the daily realized variance estimator increases as the intra-day sampling frequency increases.  (incorrect)

(d)  The GARCH-X volatility model specifies the conditional variance of day t as σt(2)  = ω + αεt(2)1 + βσt(2)1 + γRVt  where RVt  denotes the realized variance in day t.  (incorrect)

(e)  Comparing to the squared residual, the realized variance often serves as a less noisy proxy

for the unobserved variance when we evaluate volatility forecasting models.  (correct)

(f)  The realized variance process for most financial assets is persistent.  (correct)

1.2    Numerical-answer questions (12 points)

1.  Let Rt  be the arithmetic net return on this stock from day t − 1 to day t. The price of a stock (no dividend) on day t is Pt  = x. Suppose that based on some information set Ft  a forecasting model predicts the variance of stock price on day t+1 as Vart (Pt+1) = y . What is the predicted variance of the stock return Rt+1?

Answer : We have

Vart (Rt+1) = Vart  (  1) = Vart  ( ) = Vart (Pt+1) =  .

2.  Consider the following weakly stationary process

Yt  = c + ϕYt1 + θεt1 + εt ,    εt  ∼ WN(0,σ2 ),

where |ϕ| < 1. What is the value of Var(Yt+2)?

Answer : We have

Var(Yt+2) = Var(Yt ) = σ 2 .

3.  Consider a stationary GARCH(1,1) model for asset return:

Rt  = µt + εt ,    µt  = Et1(Rt )

εt  = σt νt ,    νt  i.i.d.(0, 1)

σt(2)  = ω + αεt(2) + βσt(2)1 .

Suppose that at time t, the model produce a one-step ahead volatility forecast, σt(2)+1|t  = x, what is the h-step ahead volatility forecast σt(2)+h|t?

Answer : We use the formula

σt(2)+h|t  = 2 + (α + β)h1(σt(2)+1|t 2 )

where 2  = ω/(1 − α − β).

4.  Suppose a stock return follows a normal distribution with mean µ and variance σ 2 , what is the α return VaR (Value at Risk)?

Answer : The α return VaR is

VaRα  = −µ − σΦ1(α)

where  Φ( · )  is the  cdf of standard normal  distribution.   (need  a statistical table of normal

quantiles here)

5.  Suppose we use an ARMA(1,1) model to forecast the index stock returns {Rt }: Rt+1  = c + ϕRt + θεt + εt+1 ,

where R0  = r , ε0  = 0 and E(εt+1|Ft ) = 0 for all t.  At time t = 1, we have observed R1  = α . What is the conditional mean point forecast of R2  at time t = 1?

Answer:  The conditional mean point forecast of R2  at time t = 1 is

Rˆ2  := E(R2 |F1 ) = E(c + ϕR1 + θε1 + ε2 |F1 ) = c + ϕR1 + θε1                                 (1)

under the model assumptions.  Although ε 1  is not directly observable, it can be deduced from the observed R1  because the model implies

R1  = c + ϕR0 + θε0 + ε 1       ⇒  ε 1  = R1 − c − ϕR0 − θε0  = R1 − c − ϕR0  = α − c − ϕr.   (2)

Combining (1) and (2) yields

Rˆ2  = c + ϕα + θ(α − c − ϕr).

6.  Suppose that we want to predict the 5 percent daily Value at Risk and Expected Shortfall of an index percentage return on 7 June 2022. We consider the historical simulation method using the percentage returns of this index in the last 100 days. We sort these returns from the largest to the smallest and the sorted data look like:

R(1) ,R(2) ,R(3) ,R(4) ,R(5) ,R(6) ,R(7) , . . . ,R(94) ,R(95) ,R(96) ,R(97) ,R(98) ,R(99) ,R(100) .

What is our 5 percent daily Value at Risk prediction given this data set? What is the 5 percent Expected Shortfall prediction given this data set?

Answer : 5 percent daily Value at Risk prediction is −R(96) . 5 percent daily Expected Shortfall

prediction is

100                                100

  R(i)  =   R(i) .

1.3    Short-answer questions (12 points)

1.  Explain what the half-life” of a volatility forecasting model is.  (2pt) What can you say about the model if it has a longer half-life.  (2pt)

Marking:  The multi-period ahead volatility forecasts often revert to the unconditional volatility in the long run. The half-life” is the number of periods after which the conditional variance is half way towards the unconditional variance.  (2pt)

The model with a longer half-life will have more persistent conditional variance/volatility pro- cess. The speed at which the multi-period ahead volatility forecasts revert to the unconditional volatility level will be slower.  (2pt)

It’s okay if students use GARCH model as an example to answer this question. See the discussion of the half-life of GARCH(1,1) model on p.  150 of Patton (2019).

2.  Suppose that we have obtained some estimation results of a GJR-GARCH(1,1) model: σt(2)  = ω + αεt(2)1 + βσt(2)1 + 6εt(2)11{εt1  ≤ 0}

using the returns of three assets:  AUD/USD exchange rate, S&P/ASX 200 index, and RBA cash rate.

ω         α         β          6        6 t

 

AUD/USD exchange rate

S&P/ASX 200 index

RBA cash rate

1.25

1.8

0.05

0.025 0.02 0.12

0.95

0.89

0.8

0.005

0.01

-0.07

1.2

3.5

-2.56

Please interpret the results in the above table,  and explain what you can learn about the properties of these three asset return series.  (4pt)

Marking:  From the t statistics of the 6 estimates, we can see that:

• The asymmetry parameter 6 is not significantly different from zero in the equation for the exchange rate.  This means the impacts of the positive and negative shocks (of the same magnitude in absolute values) on the future volatility are not significantly different.  (1pt)

• The asymmetry parameter 6 is significantly positive in the equation for the stock index. This indicates that negative shocks lead to higher future volatility than do positive shocks of the same magnitude the so-called leverage effect” .  (1pt)

• The asymmetry parameter 6 is significantly negative in the equation for the interest rate. This indicates that positive shocks lead to higher future volatility than do negative shocks of the same magnitude.  (1pt)

The other 1pt awards other sensible interpretations of the estimation results in the table.

3.  Explain what an autoregressive conditional duration model is (2pt), and give an example of the application of this model in financial econometrics (2pt)

Marking:  See the autoregressive conditional duration (ACD) model in Section 11.4.2 (p.  391) of Patton (2019). Note that the ACD model has a very similar structure as that of the GARCH model.

I expect most students to mention modelling the durations between financial transactions as the application of ACD models in financial econometrics, while there definitely can be other correct answers.

1.4    File-upload questions (9 points)

1.  Suppose that we want to use a generalized Mincer-Zarnowitz regression to conduct a conditional coverage test for the optimality of a one-step ahead 1% Value at Risk forecast  VaRt+1  for an asset return series {Rt+1}. Please describe how you would conduct the test by explicitly

(1) writing down the regression that you would run, (1pt)

(2)  describing how you would construct the variables in your regression, (2pt)

(3)  stating the hypothesis that you would like to test, (1pt)

(4) explaining how you would test the hypothesis.  (1pt)

Answer:  The Mincer-Zarnowitz regression for evaluating the optimality of the α-VaR forecast is based on the following condition:

Et (Hitt+1) = α

where Hitt+1  = 1{Rt+1  > VaR} and VaR is the one-period ahead optimal α-VaR forecast at time t.

To test the optimality of the a one-step ahead 1% Value at Risk forecast  VaRt+1, we should first construct the  Hit” variable itt+1  = 1{Rt+1  >  VaRt+1}.  Then, we run the generalized Mincer-Zarnowitz regression

itt+1  = α + β VaRt+1 + ut+1 ,                                                (3)

and test

H0  : α = 0.01   and   β = 0,

by using a joint test (F test or χ2  test).

Note: This question asks to use the generalized” MZ regression, so there must be some regres- sor(s) other than the constant term in the regression (3). The regressors can be more than one, and they have to be the variables known at time t.  For example, apart from  VaRt+1  used in (3), one can also include variables such as Rt , Rt1 ,  VaRt , itt  in the regression. The test is a joint test of the constant being 0.01 and the coefficient(s) of the regressor(s) being zero.

2.  Suppose that we would like to use the maximum likelihood method to estimate the intercept and AR coefficient in an AR(1) model

Yt  = ϕ0 + ϕ1 Yt1 + εt ,    εt  ∼ N(0, 1),

using the observed data Y1 , . . . ,YT . Please

(1)  derive the maximum likelihood estimators for ϕ0  and ϕ 1  with detailed intermediate steps, (3pt) and

(2)  compare the analytical form of the maximum likelihood estimators for ϕ0  and ϕ 1  with that of the OLS estimators.  (1pt)

Answer:  The AR(1) model together with the standard normal distribution of the error term implies that

Yt |Ft1  ∼ N(ϕ0 + ϕ1 Yt1 , 1).

Then, by successive conditioning, the joint density of the observations (Y1 , . . . ,YT ) is

T                                                                         T

and the log-likelihood function given realized observations (Y1 , . . . ,YT ) is

T

L(ϕ0 ,ϕ 1 ) =  (Yt ϕ0 ϕ1 Yt1)2 ,                                        (4)

t=2

where the constant term is omitted.   To derive the ML estimators,  we consider first-order conditions (FOC):

 = (Yt − ϕ0 − ϕ1 Yt1) = 0,

 = (Yt − ϕ0 − ϕ1 Yt1)Yt1  = 0.

From (5) we can deduce that

T                                T

(T 1)0(MLE) = Yt 1(M)LE  Yt1 ,

t=2                            t=2

which implies that

0(MLE) = Yt 1(M)LEYt1 ,

where Yt  :=   Yt  and Yt1  :=    .

From (6) we can deduce that

T                                           T                                     T

 Yt Yt1 0(MLE)  Yt1 1(M)LE  Yt1  = 0

t=2                                      t=2                                 t=2

Plugging (7) into (8) yields

 (T 1)Yt · Yt1 1(M)LE  ( Yt1 (T 1)Yt(2)1) = 0,

which gives

1(M)LE  = (Yt1  Yt1)(Yt  Yt )

(Yt1  Yt1)2            .

We can see from (7) and (9) that the analytical form of the MLE is the same as that of the OLS. In fact, maximizing the log-likelihood function in (4) is equivalent to minimizing the sum of squared residuals of the autoregression.